导数压轴题

1.导数压轴题精选

https://wenku.baidu.com/view/4302e31da517866fb84ae45c3b3567ec102ddcc9.html?rec_flag=default&word=%E5%AF%BC%E6%95%B0%E5%8E%8B%E8%BD%B4%E9%A2%98&fr=pc_newview_relate-1001_1-1-wk_rec_doc2-1001_1-3-4302e31da517866fb84ae45c3b3567ec102ddcc9

2.浙江高考导数压轴100题

https://wenku.baidu.com/view/7a8ef5c10708763231126edb6f1aff00bed57019.html?rec_flag=default&word=%E5%AF%BC%E6%95%B0%E5%8E%8B%E8%BD%B4%E9%A2%98&fr=pc_newview_relate-1001_1-1-wk_rec_doc2-1001_1-3-7a8ef5c10708763231126edb6f1aff00bed57019

https://wenku.baidu.com/view/70d8af104a2fb4daa58da0116c175f0e7cd119e4.html

 https://wenku.baidu.com/view/bf4bee43bcd5b9f3f90f76c66137ee06eff94ec5.html

egin{problem}
(2005年高考全国1卷)
egin{enumerate}
item[(1)] 设函数$f(x)=x log _{2} x+(1-x) log _{2}(1-x)\,(0<x<1)$,求$f(x)$的最小值;

item[(2)] 设正数$p_1,p_2,p_3,cdots,p_{2^n}$满足$p_{1}+p_{2}+p_{3}+cdots+p_{2^{n}}=1$,证明:
[p_{1} log _{2} p_{1}+p_{2} log _{2} p_{2}+p_{3} log _{2} p_{3}+cdots+p_{2^{n}} log _{2} p_{2^{n}} geq -n.]

item[(3)] (熵不等式)设 $p_1,p_2,p_3,cdots,p_n$和 $q_1,q_2,q_3,cdots,q_n$均为正数且$sum_{k=1}^{n}p_k=sum_{k=1}^{n}q_k$.求证:
[sum_{k=1}^{n}p_kln p_kgeq sum_{k=1}^{n}p_kln q_k.]
end{enumerate}
end{problem}
egin{solution}
egin{enumerate}
item[(1)] 对函数$f(x)$求导数:
egin{align*}
f'(x) &=left(x log _{2} x ight)'+left[(1-x) log _{2}(1-x) ight]' \
&=log _{2} x-log _{2}(1-x)+frac{1}{ln 2}-frac{1}{ln 2} \ &=log _{2} x-log _{2}(1-x).
end{align*}
于是$f'left(frac{1}{2} ight)=0$.

当$x<frac{1}{2}$时, $f'(x)=log _{2} x-log _{2}(1-x)<0$, $f(x)$在区间$left(0,frac{1}{2} ight)$是减函数;当$x>frac{1}{2}$时, $f'(x)=log _{2} x-log _{2}(1-x)>0$, $f(x)$在区间$left(frac{1}{2},1 ight)$是增函数.

所以$f(x)$在$x=frac{1}{2}$时取得最小值, $fleft(frac{1}{2} ight)=-1$.

item[(2)] extbf{证法一:}用数学归纳法证明.

egin{enumerate}
item[(i)] 当$n=1$时,由(I)知命题成立.

item[(ii)] 假定当$n=k$时命题成立,即若正数$p_1,p_2,p_3,cdots,p_{2^k}$满足$p_{1}+p_{2}+p_{3}+cdots+p_{2^{k}}=1$,则
[p_{1} log _{2} p_{1}+p_{2} log _{2} p_{2}+p_{3} log _{2} p_{3}+cdots+p_{2^{k}} log _{2} p_{2^{k}} geq -k.]

当$n=k+1$时,若正数$p_1,p_2,p_3,cdots,p_{2^{k+1}}$满足$p_{1}+p_{2}+p_{3}+cdots+p_{2^{k+1}}=1$,令
[x=p_{1}+p_{2}+cdots+p_{2^{k}}, q_{1}=frac{p_{1}}{x}, q_{2}=frac{p_{2}}{x}, cdots, q_{2^{k}}=frac{p_{2^k}}{x}.]
则$q_1,q_2,q_3,cdots,q_{2^k}$为正数,且$q_{1}+q_{2}+cdots+q_{2^{k}}=1$.

由归纳假定知$q_{1} log _{2} q_{1}+q_{2} log _{2} q_{2}+cdots+q_{2^k} log _{2} q_{2^k} geq -k$.
egin{align*}
p_{1} log _{2} p_{1}+p_{2} log _{2} p_{2}+cdots+p_{2^k} log _2 p_{2^{k}} &=xleft(q_{1} log _{2} q_{1}+q_{2} log _{2} q_{2}+cdots+q_{2^k} log _{2} q_{2^k}+log _{2} x ight)\
&geq x(-k)+xlog_2 x, ag*{(1)}
end{align*}
同理,由$p_{2^{k}+1}+p_{2^{k}+2}+cdots+p_{2^{k+1}}=1-x$,可得
[p_{2^{k}+1} log _{2^{k}+1}+cdots+p_{2^{k+1}} log _{2^{k}+1}geq (1-x)(-k)+(1-x). log _{2}(1-x). ag*{(2)}]
综合(1)、(2)两式
egin{align*}
p_{1} log _{2} p_{1}+& p_{2} log _{2} p_{2}+cdots+p_{2^{k+1}} log _{2} p_{2^{k+1}} \
& geq [x+(1-x)](-k)+x log _{2} x+(1-x) log _{2}(1-x) \
& geq -(k+1).
end{align*}
即当$n=k+1$时命题也成立.
end{enumerate}
根据(i)、(ii)可知对一切正整数$n$命题成立.

extbf{证法二:}令函数$g(x)=x log _{2} x+(c-x) log _{2}(c-x)$ (常数$c>0,xin (0,c)$),那么
[g(x)=cleft[frac{x}{c} log _{2} frac{x}{c}+left(1-frac{x}{c} ight) log _{2}left(1-frac{x}{c} ight)+log _{2} c ight],]
利用(I)知,当$frac{x}{c}=frac{1}{2}$ (即$x=frac{c}{2}$)时,函数$g(x)$取得最小值.

于是对任意$x_1>0,x_2>0$,都有
egin{align*}
x_{1} log _{2} x_{1}+x_{2} log _{2} x_{2} & geq 2 cdot frac{x_{1}+x_{2}}{2} log _{2} frac{x_{1}+x_{2}}{2} \ &=left(x_{1}+x_{2} ight)left[log _{2}left(x_{1}+x_{2} ight)-1 ight]. ag*{(1)}
end{align*}

下面用数学归纳法证明结论.

egin{enumerate}
item[(i)] 当$n=1$时,由(I)知命题成立.

item[(ii)] 假定当$n=k$时命题成立,即若正数$p_1,p_2,p_3,cdots,p_{2^k}$满足$p_{1}+p_{2}+p_{3}+cdots+p_{2^{k}}=1$,有
[p_{1} log _{2} p_{1}+p_{2} log _{2} p_{2}+cdots+p_{2^{k}} log _{2} p_{2^{k}} geq -k.]

当$n=k+1$时, $p_1,p_2,p_3,cdots,p_{2^{k+1}}$满足$p_{1}+p_{2}+p_{3}+cdots+p_{2^{k+1}}=1$,令
[H=p_{1} log _{2} p_{1}+p_{2} log _{2} p_{2}+cdots+p_{2^{k+1}-1} log _{2} p_{2^{k+1}-1}+p_{2^{k+1}} log _{2} p_{2^{k+1}},]
由(1)得到
[H geqleft(p_{1}+p_{2} ight)left[log _{2}left(p_{1}+p_{2} ight)-1 ight]+cdots+left(p_{2^{k+1}-1}+p_{2^{k+1}} ight)left[log _{2}left(p_{2^{k+1}-1}+p_{2^{k+1}} ight)-1 ight],]
因为$left(p_{1}+p_{2} ight)+cdots+left(p_{2^{k+1}-1}+p_{2^{k+1}} ight)=1$,由归纳法假设
[left(p_{1}+p_{2} ight) log _{2}left(p_{1}+p_{2} ight)+cdots+left(p_{2^{k+1}-1}+p_{2^{k+1}} ight) log _{2}left(p_{2^{k+1-1}}+p_{2^{k+1}} ight) geq -k,]
得到
[H geq -k-left(p_{1}+p_{2}+cdots+p_{2^{k+1}-1}+p_{2^{k+1}} ight)=-(k+1).]
即当$n=k+1$时命题也成立.
end{enumerate}
所以对一切正整数$n$命题成立.

item[(3)] 利用不等式$tln tgeq t-1,t>0$.事实上,
egin{align*}
sum_{k=1}^{n} p_{k} ln p_{k}-sum_{k=1}^{n} p_{k} ln q_{k} &=sum_{k=1}^{n} p_{k} ln frac{p_{k}}{q_{k}
=sum_{k=1}^{n} q_{k} frac{p_{k}}{q_{k}} ln frac{p_{k}}{q_{k}}} \
&geq sum_{k=1}^{n} q_{k}left(frac{p_{k}}{q_{k}}-1 ight)=sum_{k=1}^{n}left(p_{k}-q_{k} ight)=0.
end{align*}
end{enumerate}
end{solution}

egin{problem}
(2004年高考全国2卷)已知函数$f(x)=ln (1+x)-x, g(x)=x ln x$.
egin{enumerate}
item[(1)] 求函数$f(x)$的最大值;

item[(2)] 设$0<a<b$,证明: $0<g(a)+g(b)-2 gleft(frac{a+b}{2} ight)<(b-a) ln 2$.
end{enumerate}
end{problem}
egin{solution}
egin{enumerate}
item[(1)] 函数$f(x)$的定义域是$(-1,infty),f'(x)=frac{1}{1+x}-1$.令$f'(x)=0$,解得$x=0$.当$-1<x<0$时, $f'(x)>0$;当$x>0$时, $f'(x)<0$,又$f(0)=0$,故当且仅当$x=0$时, $f(x)$取得最大值,最大值是$0$.

item[(2)] extbf{证法一.}
[g(a)+g(b)-2 gleft(frac{a+b}{2} ight)=a ln a+b ln b-(a+b) ln frac{a+b}{2}=a ln frac{2 a}{a+b}+b ln frac{2 b}{a+b}.]
由(1)的结论知$ln (1+x)-x<0$ ($x>-1$且$x eq 0$),由题设$0<a<b$,得$frac{b-a}{2 a}>0,-1<frac{a-b}{2 b}<0$,因此
[ln frac{2 a}{a+b}=-ln left(1+frac{b-a}{2 a} ight)>-frac{b-a}{2 a},quad ln frac{2 b}{a+b}=-ln left(1+frac{a-b}{2 b} ight)>-frac{a-b}{2 b}.]
所以
[a ln frac{2 a}{a+b}+b ln frac{2 b}{a+b}>-frac{b-a}{2}-frac{a-b}{2}=0.]

[frac{2 a}{a+b}<frac{a+b}{2 b}, quad a ln frac{2 a}{a+b}+b ln frac{2 b}{a+b}<a ln frac{a+b}{2 b}+b ln frac{2 b}{a+b}=(b-a) ln frac{2 b}{a+b}<(b-a) ln 2.]
综上, $0<g(a)+g(b)-2 gleft(frac{a+b}{2} ight)<(b-a) ln 2$.


extbf{证法二.} $g(x)=xln x,g'(x)=ln x+1$,设$F(x)=g(a)+g(x)-2 gleft(frac{a+x}{2} ight)$,则$F^{prime}(x)=g^{prime}(x)-2left[gleft(frac{a+x}{2} ight) ight]^{prime}=ln x-ln frac{a+x}{2}$.当$0<x<a$时$F'(x)<0$,因此$F(x)$在$(0,a)$内为减函数;当$x>a$时$F'(x)>0$,因此$F(x)$在$(a,+infty)$上为增函数.从而,当$x=a$时, $F(x)$有极小值$F(a)$.因为$F(a)=0,b>a$,所以$F(b)>0$,即$0<g(a)+g(b)-2 gleft(frac{a+b}{2} ight)$.

设$G(x)=F(x)-(x-a) ln 2$,则$G^{prime}(x)=ln x-ln frac{a+x}{2}-ln 2=ln x-ln (a+x)$.当$x>0$时$G'(x)<0$,因此$G(x)$在$(0,+infty)$上为减函数.因为$G(a)=0,b>a$,所以$G(b)<0$,即$g(a)+g(b)-2 gleft(frac{a+b}{2} ight)<(b-a) ln 2$.
end{enumerate}
end{solution}

documentclass[a4paper]{ctexart}

usepackage[margin=1in]{geometry}

itle{导数与圆锥曲线练习题}


%author{高思一对一高端VIP部:曾熊quad 老师}

date{2020年9月24日}

usepackage{amsmath,ntheorem,amssymb,amsfonts}

heorembodyfont{upshape}
heoremseparator{.}
% heoremstyle{break}
ewtheorem{problem}{问题}

everymath{displaystyle}%%公式都不缩小
allowdisplaybreaks[4]%%公式跨页


ewenvironment{solution}{par oindent extbf{{解:}}}

egin{document}
maketitle


section{导数}

egin{problem}
已知函数$f(x)=e^x-ax^2\,(ain mathbb{R})$.
egin{enumerate}
item[(I)] 若曲线$y=f(x)$在$(1,f(1))$处的切线与$x$轴平行,求$a$;

item[(II)] 已知$f(x)$在$[0,1]$上的最大值不小于$2$,求$a$的取值范围;

item[(III)] 写出$f(x)$所有可能的零点个数及相应的$a$的取值范围.
end{enumerate}
end{problem}
%egin{solution}

%end{solution}

egin{problem}
设函数$f(x)=e^x-1-x-ax^2$.

(1)若$a=0$,求$f(x)$的单调区间;

(2)若当$xgeq 0$时$f(x)geq 0$,求$a$的取值范围.
end{problem}

拓展:函数不等式: $e^xgeq x+1\, (xin mathbb{R}),ln (x+1)leq x\,(x>-1)$与$sin x leq xleq an x\,left(0<x<frac{pi}{2} ight)$.

泰勒公式:
[e^x=1+x+frac{x^2}{2!}+frac{x^3}{3!}+cdots+frac{x^n}{n!}+cdots]


egin{problem}
%(2020年全国I卷理数)
己知函数$f(x)=e^x+ax^2-x$.

(1)当$a=1$时,讨论$f(x)$的单调性;

(2)当$xgeqslant 0$时, $f(x)geqslant frac{1}{2}x^3+1$,求$a$的取值范围.
end{problem}


egin{problem}
%(2017年全国III卷)
已知函数$f(x)=x-1-aln x$.

(1)若$f(x)geqslant 0$,求$a$的值;

(2)设$m$为整数,且对于任意正整数$n$, $left( 1+frac{1}{2} ight)left( 1+frac{1}{2^2} ight)cdots left( 1+frac{1}{2^n} ight)<m$,求$m$最小值.
end{problem}

egin{problem}
已知函数$f(x)=xln x$.

(1)求函数$f(x)$的最小值;

(2)若对一切$xin (0,+infty)$,都有$f(x)leqslant x^2-ax +2$恒成立,求实数$a$的取值范围;

(3)试判断函数$y=ln x$与$g(x)=frac{1}{e^x}-frac{2}{ex}$的图象是否有公共点?若有,求出公共点的个数;若无,请说明理由.
end{problem}

egin{problem}
%(2016年全国I卷理数)
已知函数$f(x)=(x-2)e^x+a(x-1)^2$有两个零点.

(I)求$a$的取值范围;

(II)设$x_1,x_2$是$f(x)$的两个零点,求证: $x_1+x_2<2$.
end{problem}

egin{problem}
%(2017年北京理数)
已知函数$f(x)=e^xcos x-x$.

(I)求曲线$y=f(x)$在点$(0,f(0))$处的切线方程;

(II)求函数$f(x)$在区间$left[0,frac{pi}{2} ight]$上的最大值和最小值.
end{problem}

egin{problem}
%(2014年北京理数)
已知函数$f(x)=xcos x-sin x,xinleft[0,frac{pi}{2} ight]$.

(1)求证: $f(x)leqslant 0$;

(2)若$a<frac{sin x}{x}<b$在$left(0,frac{pi}{2} ight)$上恒成立,求$a$的最大值与$b$的最小值.
end{problem}

egin{problem}
%(2010年湖北理数)
已知函数$f(x)=ax+frac{b}{x}+c\,(a>0)$的图象在点$(1,f(1))$处的切线方程为$y=x-1$.

(1)用$a$表示出$b$、$c$;

(2)若不等式$f(x)>ln x$在$xin (1,+infty)$上恒成立,求实数$a$的取值范围;

(3)证明不等式: $1+frac{1}{2}+frac{1}{3}+cdots +frac{1}{n}> ln (n+1)+frac{n}{2(n+1)}\,(nin mathbb{N}^ast)$.
end{problem}


egin{problem}
设函数$f(x)=frac{1-x}{ax}+ln x$在$[1,+infty)$上是增函数.

(1)求正实数$a$的取值范围;

(2)设$b>0,a>1$,求证: $frac{1}{a+b}<ln frac{a+b}{b}<frac{a+b}{b}$.
end{problem}


egin{problem}
已知函数$f(x)=2ln x+frac{1}{x}-mx\,(min mathbb{R})$.

(I)当$m=-1$时,求曲线$y=f(x)$在点$(1,f(1))$处的切线方程;

(II)若$f(x)$在$(0,+infty)$上为单调递减,求$m$的取值范围;

(III)设$0<a<b$,求证: $frac{2}{a+b}<frac{ln b-ln a}{b-a}<frac{1}{sqrt{ab}}$.
end{problem}

拓展: (对数平均值不等式)设$0<a<b$,则
$$a<frac{2}{frac{1}{a}+frac{1}{b}}
<sqrt{ab}<frac{b-a}{ln b-ln a}<frac{a+b}{2}<sqrt{frac{a^2+b^2}{2}}.$$

%https://www.sohu.com/a/279335998_120009991


egin{problem}
%(2004年高考全国2卷)
已知函数$f(x)=ln (1+x)-x, g(x)=x ln x$.
egin{enumerate}
item[(1)] 求函数$f(x)$的最大值;

item[(2)] 设$0<a<b$,证明: $0<g(a)+g(b)-2 gleft(frac{a+b}{2} ight)<(b-a) ln 2$.
end{enumerate}
end{problem}

%2015年广东

egin{problem}
%(2019年全国II卷理数)
已知函数$f(x)=ln x-frac{x+1}{x-1}$.

(1)讨论$f(x)$的单调性,并证明$f(x)$有且仅有两个零点;

(2)设$x_0$是$f(x)$的一个零点,证明曲线$y=ln x$在点$A(x_0,ln x_0)$处的切线也是曲线$y=e^x$的切线.
end{problem}

egin{problem}
%(2005年湖南理数)
已知函数$f(x)=ln x,g(x)=frac{1}{2}ax^2+bx,a eq 0$.

(I)若$b=2$,且函数$h(x)= f(x)-g(x)$存在单调递减区间,求$a$的取值范围.

(II)设函数$f(x)$的图象$C_1$与函数$g(x)$的图象$C_2$交于点$P,Q$,过线段$PQ$的中点作$x$轴的垂线分别交$C_1,C_2$于点$M,N$.证明$C_1$在点$M$处的切线与$C_2$在点$N$处的切线不平行.
end{problem}

egin{problem}
%(云南师大附中2019-2020学年高三适应性)
已知$f(x)=e^x,g(x)=ln x$,若点$A$为函数$f(x)$上的任意一点,点$B$为函数$g(x)$上的任意一点.

(1)求$A,B$两点之间距离的最小值;

(2)若$A,B$为函数$f(x)$与函数$g(x)$公切线的两个切点,求证:这样的点$B$有且仅有两个,且满足条件的两个点$B$的横坐标互为倒数.
end{problem}


section{圆锥曲线}

egin{problem}
%(2011年北京)
已知椭圆$G:frac{x^2}{4}+y^2=1$,过点$(m,0)$作圆$x^2+y^2=1$的切线$l$交椭圆$G$于$A,B$两点.

(I)求椭圆$G$的焦点坐标和离心率;

(II)将$|AB|$表示为$m$的函数,并求$|AB|$的最大值.
end{problem}


egin{problem}
%(2009年北京)
已知双曲线$C:frac{x^2}{a^2}-frac{y^2} {b^2}=1\,(a>0,b>0)$的离心率为$sqrt{3}$,右准线方程为$x=frac{sqrt{3}}{3}$.

(I)求双曲线$C$的方程;

(II)设直线$l$是圆$O:x^2+y^2=2$上动点$P(x_0,y_0)\,(x_0y_0 eq 0)$处的切线, $l$与双曲线$C$交于不同的两点$A,B$,证明$angle AOB$的大小为定值.
end{problem}

egin{problem}
%(2011年华约)
已知双曲线$C:frac{x^2}{a^2}-frac{y^2} {b^2}=1\,(a>0,b>0)$, $F_1,F_2$分别为$C$的左右焦点. $P$为$C$右支上一点,且使$angle F_1PF_2=frac{pi}{3}$,又$ riangle F_1PF_2$的面积为$3sqrt{3}a^2$.

(I)求$C$的离心率$e$;

(II)设$A$为$C$的左顶点, $Q$为第一象限内$C$上的任意一点,问是否存在常数$lambda\,(lambda>0)$,使得
$angle QF_2A=lambda angle QAF_2$恒成立.若存在,求出$lambda$的值;若不存在,请说明理由.
end{problem}

egin{problem}
%(2020年全国I卷)
已知$A,B$分别为椭圆$E: frac{x^2}{a^2}+y^2=1\, (a>1)$的左、右顶点, $G$为$E$上顶点, $overrightarrow{AG}cdot overrightarrow{GB}=8$. $P$为直线$x=6$上的动点, $PA$与$E$的另一交点为$C$, $PB$与$E$的另一交点为$D$.

(1)求$E$的方程;

(2)证明:直线$CD$过定点.
end{problem}

egin{problem}
%(2020年新高考I卷)
已知椭圆$C:frac{x^2}{a^2}+frac{y^2}{b^2}=1\,(a>b> 0)$的离心率为$frac{sqrt{2}}{2}$,且过点$A(2,1)$.

(1)求$C$的方程.

(2)点$M,N$在$C$上,且$AMperp AN,ADperp MN$, $D$为垂足,证明:存在定点$Q$,使得$|DQ|$为定值.
end{problem}

egin{problem}
%(2020年北京市海淀区高考数学一模试卷)
已知椭圆$C:frac{x^2}{a^2}+frac{y^2}{b^2}=1\,(a>b>0)$的离心率为$frac{sqrt{3}}{2}$, $A_1(-a,0),A_2(a,0),B(0,b)$, $ riangle A_1BA_2$的面积为$2$.

(I)求椭圆$C$的方程;

(II)设$M$是椭圆$C$上一点,且不与顶点重合,若直线$A_1B$与直线$A_2M$交于点$P$,直线$A_1M$与直线$A_2B$交于点$Q$.求证: $ riangle BPQ$为等腰三角形.
end{problem}


egin{problem}
(2017年北京市东城区高三第一学期期末)
已知椭圆$C:frac{x^2}{a^2}+frac{y^2}{b^2}=1\,(a>b>0)$经过点$M(2,0)$,离心率为$frac{1}{2}$. $A,B$是椭圆$C$上两点,且直线$OA,OB$的斜率之积为$-frac{3}{4}$, $O$为坐标原点.

(I)求椭圆$C$的方程;

(II)若射线$OA$上的点$P$满足$|PO|=3|OA|$,且$PB$与椭圆交于点$Q$,求$frac{|BP|} {|BQ|}$的值.
end{problem}
%https://wenku.baidu.com/view/ccabfd4c7275a417866fb84ae45c3b3567ecddaf.html


egin{problem}
(2020年北京高考)
已知椭圆$C:frac{x^2}{a^2}+frac{y^2}{b^2}=1$过点$A(-2,-1)$,且$a=2b$.

(I)求椭圆$C$的方程;

(II)过点$B(-4,0)$的直线$l$交椭圆$C$于点$M,N$,直线$MA,NA$分别交直线$x=-4$于点$P,Q$.求$frac{|PB|}{|BQ|}$的值.
end{problem}


egin{problem}
设$a_1,a_2,a_3,cdots,a_{2019},a_{2020}$是正整数 $1$到$2020$的一个排列,且$x_1=a_1$, $x_2=|a_2-x_1|$, $x_3=|a_3-x_2|$, $cdots$, $x_{2019}=|a_{2019}-x_{2018}|$, $x_{2020}=|a_{2020}-x_{2019}|$.
试求出$x_{2020}$的最小值和最大值.
end{problem}

%(2020年北京高考)已知$alpha,etain mathbb{R}$,则"存在$kin mathbb{Z}$使得$a=kpi +(-1)^keta$"是"$sinalpha=sineta$"的

%A.充分而不必要条件 B.必要而不充分条件 C.充分必要条件 D.既不充分也不必要条件

egin{problem}
已知$n$个不同的数$x_1,x_2,x_3,cdots,x_n$是正整数$1,2,cdots,n$的任意一个排列,试求$|x_1-1|+|x_2-2|+cdots+|x_n-n|$的最小值.
end{problem}

八皇后-高斯

骑士遍历-欧拉

egin{problem}

end{problem}

end{document}

原文地址:https://www.cnblogs.com/Eufisky/p/13704241.html